LSAT and Law School Admissions Forum

Get expert LSAT preparation and law school admissions advice from PowerScore Test Preparation.

User avatar
 Dave Killoran
PowerScore Staff
  • PowerScore Staff
  • Posts: 5852
  • Joined: Mar 25, 2011
|
#41158
Complete Question Explanation
(The complete setup for this game can be found here: lsat/viewtopic.php?t=11758)

The correct answer choice is (E)

This is the type of question where one might expect that J and K would be the correct answer (if J and K were in the same class as F, there would not be a third student for the level 1 class). However, that answer does not appear among the five lettered choices. Instead, we will need to find a different pair that creates a problem.

Answer choice (A) is incorrect because F, H, and N can be in the level 3 class, leaving L, M, and J or K for the level 2 class, and I, G and K or J for the level 1 class. Thus, as this student pair produces a workable solution, this answer choice is incorrect.

Answer choice (B) is incorrect because J, L, and F can be in the level 2 class, leaving M, H and N in the level 3 class, and I, G and K for the level 1 class. Thus, as this student pair produces a workable solution, this answer choice is incorrect.

Answer choice (C) is incorrect because K, L, and F can be in the level 3 class, leaving M, H and N for the level 2 class, and I, G and J for the level 1 class. Thus, as this student pair produces a workable solution, this answer choice is incorrect.

Answer choice (D) is incorrect because J, M, and F can be in the level 2 class, leaving L, H and N in the level 3 class, and I, G and K for the level 1 class. Thus, as this student pair produces a workable solution, this answer choice is incorrect.

In answer choice (E), placing F with L and M leaves only H and N for the level 3 class. Since each level must have three students, the scenario in answer choice (E) cannot occur, and therefore (E) is correct.
 Lulu
  • Posts: 8
  • Joined: Oct 06, 2019
|
#70931
Hi,

"In answer choice (E), placing F with L and M leaves only H and N for the level 3 class. Since each level must have three students, the scenario in answer choice (E) cannot occur, and therefore (E) is correct."

But I thought we could put "K" in level 3, creating the following scenario that (E) can occur:

Level 1-I, G, J
Level 2-F, L, M
Level 3-K, H, N

OR

we could put "K" in level 2, creating another scenario that (E) can occur:

Level 1 - I, G, J
Level 2 - K, H, N
Level 3 - F, L, M

I'd appreciate it if you could clarify.

Thanks.
User avatar
 Dave Killoran
PowerScore Staff
  • PowerScore Staff
  • Posts: 5852
  • Joined: Mar 25, 2011
|
#70939
Hi lulu,

Thanks for the question! Usually when you see an answer or explanation that is counter to a hypothetical you have created, the best approach is to check it against the original rules to see if you made an error along the way. In this case, let's look at each scenario you posed and see where you ran into problems:
Lulu wrote:But I thought we could put "K" in level 3, creating the following scenario that (E) can occur:

Level 1-I, G, J
Level 2-F, L, M
Level 3-K, H, N
This scenario most obviously violates the last rule, which states that "Kathy scores higher than both Fred and Laura." In your scenario, K is in the level 3 class, and F and L are both in the level 2 class, which cannot occur since the level 3 class is the three lowest scorers.

Lulu wrote:we could put "K" in level 2, creating another scenario that (E) can occur:

Level 1 - I, G, J
Level 2 - K, H, N
Level 3 - F, L, M
This scenario violates the fourth rule, which states that "Mike scores higher than Hilary." In your scenario, M is in the level 3 class, and H is in the level 2 class, which cannot occur since the level 3 class is the three lowest scorers.

Please let me know if that helps. Thanks!
 saygracealways
  • Posts: 34
  • Joined: Apr 09, 2020
|
#74835
Hi Powerscore,

I answered this question correctly, but am wondering if there is a way that I can narrow down the choices more quickly instead of eliminating all the Could be True possibilities before arriving at the correct answer (E)? For instance, can I use the inference that either J/K must be in level 1 (like in question #4) to narrow down the answer choices and save time?

Thank you!
User avatar
 KelseyWoods
PowerScore Staff
  • PowerScore Staff
  • Posts: 1079
  • Joined: Jun 26, 2013
|
#74855
Hi saygracealways!

Unfortunately, for this question you're going to have to go through each of the answer choices. As mentioned in the explanation, "J & K" would be a great answer here, but it's not the choice they end up giving us. So our inference about J & K is not going to be very helpful. There will always be some LG questions where you just have to kind of go through the answer choices and see what they've given you. But hopefully if you have a good diagram and a strong understanding of how to read the diagram, you can get through the answer choices fairly quickly!

Hope this helps!

Best,
Kelsey
 saygracealways
  • Posts: 34
  • Joined: Apr 09, 2020
|
#74867
Yes it does, thank you! :-D
 kmaragh
  • Posts: 8
  • Joined: Jun 17, 2020
|
#77036
Can someone please explain how to answer number 5? I'm having a hard time trying to understand how to analyze which students can not be in the same class as Fred.
User avatar
 Stephanie Oswalt
PowerScore Staff
  • PowerScore Staff
  • Posts: 811
  • Joined: Jan 11, 2016
|
#77076
Hi kmaragh,

Thanks for the post. I have moved your question to the thread discussing #5. Please review the above explanation and discussion, and let us know if that helps, or if you still have additional questions. :)

Thanks!
User avatar
 marie216
  • Posts: 2
  • Joined: Dec 27, 2020
|
#82777
Hi. I answered this question correctly by simply going through the possible answer choices and came here to look for a faster way to complete this question. However, after reading the explanation for Q#4 I'm wondering why we cannot apply the same framework here. There, the explanation says the correct answer choice is likely to include J or K because we need to address which of J and K will be in the level 1 class. That makes it a lot easier to find the correct answer choice because it leads you to the most likely correct answer which you can quickly verify without having to go through all of the options. Why can't we apply the same logic here? Is there any other similar logic that could point us to the direction of one of the answer choices to make this problem go faster?
User avatar
 KelseyWoods
PowerScore Staff
  • PowerScore Staff
  • Posts: 1079
  • Joined: Jun 26, 2013
|
#82820
Hi Marie!

We're always at the mercy of the answer choices that the LSAT makers give us. If they had given us J and K as an option, that would have been great! Unfortunately, with this question they decided to go for something a little less obvious which makes it difficult to prephrase this question specifically. In LG, we're not always going to be able to prephrase the answer choice specifically and sometimes we will just have to test out all of the answer choices they give us. Some questions are designed to be like this and there are not always shortcuts and quick inferences that will help us get around that--the test makers really know what they're doing! But that doesn't mean that we totally have to play the test makers' game. Look at what they're doing here: they gave us a question where we might have to try out each answer choice and the correct answer choice ends up being (E)--that's mean! But there's no rule that says when trying out answer choices you have to start with (A), then (B), then (C), etc. When I encounter a question where I think I'm going to have to try out a bunch of different answer choices, I first read through all of the answer choices without diagramming anything. Sometimes there are answer choices I can eliminate immediately because they clearly violate a rule and I don't need to diagram them to see that. Sometimes the correct answer jumps out at me and I don't need to diagram anything! But if there are still multiple answer choices I think I might want to diagram, I don't just start with whichever one was listed first. I start with the one that I think has the most likely chance of being correct because it involves a really restrictive rule or principle in the game. That way, I'm going through the answer choices in an efficient way rather than just mindlessly diagramming each answer choice in order.

Another thing worth remembering--hindsight is always 20/20. Sometimes when we're reviewing a question it's easy to say "Oh, if I had just made this inference I could have answered this question faster!" But that's not always fair. Sometimes there are big, obvious inferences that we miss that we should have gotten and we can work on making similar inferences in the future. But sometimes there are smaller inferences that sure, maybe we could have realized before we got to the answer choices, but in reality it would have taken us longer to come up with that inference on our own just by staring at the global and local rules than it did to just go through the answer choices and see what happens. Be flexible in your approach here. Not every LG question is going to work out the same way or benefit from the same strategy! :)

Hope this helps!

Best,
Kelsey

Get the most out of your LSAT Prep Plus subscription.

Analyze and track your performance with our Testing and Analytics Package.